LSAT and Law School Admissions Forum

Get expert LSAT preparation and law school admissions advice from PowerScore Test Preparation.

 Administrator
PowerScore Staff
  • PowerScore Staff
  • Posts: 8916
  • Joined: Feb 02, 2011
|
#40230
Complete Question Explanation
(The complete setup for this game can be found here: lsat/viewtopic.php?t=8561)

The correct answer choice is (B)

If one of the families owned both G and I, that family must have been the Williamses. This is because the Yandells can only own one of the buildings, and the Trents cannot own both G and I. If they did, then neither the Trents would own S (they already own G and I—the maximum number of buildings allowed), nor the Yandells would own I (the Trents would). This scenario would violate the last rule of the game, and therefore cannot be true. Consequently, the Williamses must own both G and I, which immediately eliminates answer choices (A) and (E).

Just because the Williamses own both G and I does not mean that they do not also own another building. The local condition allows for either distribution to work, which is clearly shown in Templates 2A and 2B. If the Williamses own both G and I, then the Yandells do not own I. To comply with the last rule, we must ensure that the Trents own S, which eliminates answer choice (D).

The full range of solutions is shown below, in accordance with Templates 2A and 2B.
PT73_Game_#3_#17_diagram 1.png
Answer choice (A) is incorrect, because the Williamses own G.

Answer choice (B) is the correct answer choice, as shown in Template 2A.

Answer choice (C) is incorrect, because F is owned by either the Trents or the Yandells.

Answer choice (D) is incorrect, because the Trents own S.

Answer choice (E) is incorrect, because the Williamses own I.
You do not have the required permissions to view the files attached to this post.

Get the most out of your LSAT Prep Plus subscription.

Analyze and track your performance with our Testing and Analytics Package.